LSAT and Law School Admissions Forum

Get expert LSAT preparation and law school admissions advice from PowerScore Test Preparation.

 Administrator
PowerScore Staff
  • PowerScore Staff
  • Posts: 8916
  • Joined: Feb 02, 2011
|
#61046
Please post your questions below!
 Peterh12
  • Posts: 2
  • Joined: Dec 20, 2018
|
#61299
Hi there,

I did this question wrong in the test, after review I still cannot summarize why answer A is correct. I think the flaw in the question stem is like: if A and B together will sufficiently cause C, and if C is failed, then theoretically it might because A failed, or B failed, or both. But the conclusion of question stem just arbitrarily concluded that it must be B failed.

Does my logic right?

Thanks
 James Finch
PowerScore Staff
  • PowerScore Staff
  • Posts: 943
  • Joined: Sep 06, 2017
|
#61316
Hi Peter,

You're correct, the issue in the stimulus is that it gives the conditional:

If S and H :arrow: GA, which yields the contrapositive :

GA :arrow: S or H

and concludes GA :arrow: H, arbitrarily excluding the possibility of S.

(Both S and H claim to have supported the proposal)

As a parallel flaw question, then, we're looking for the answer choice that does the same thing. The tricky part is that we aren't given an answer choice that does that looks exactly like the stimulus, at least in terms of how the conditional reasoning is presented. So in that case, we have to go back to what exactly the error in reasoning was in the stimulus. In this case, the error was that one out of two evident possibilities was chosen, without any reason given for why it would be correct and the other one incorrect.

This is exactly what happens in (A), albeit in a different conditional form to that of the stimulus. Instead we're given the following conditional:

TV Correct :arrow: Accident on Aylmer :arrow: Morgan Saw Accident Kitchen Window :arrow: Newspaper Correct

The problem is that there is no reason to believe that the TV news is correct, and not the other way around: it is also possible, based on what is given in the answer choice, that the newspaper is correct and thus the TV news is incorrect (the contrapositive of the conditional chain given). But as no reason/evidence is given for this, it's simply assumed, then we can't conclude that either possibility is correct. So the fundamental flaw is the same as in the stimulus, even if it is presented differently.

Hope this clears things up!
 LawyerBae24
  • Posts: 3
  • Joined: May 13, 2020
|
#75970
I narrowed down to A and E on this question and chose E. Is E incorrect because it doesn’t premise a ultimatum that one or the other is incorrect?
 Rachael Wilkenfeld
PowerScore Staff
  • PowerScore Staff
  • Posts: 1358
  • Joined: Dec 15, 2011
|
#76108
Hi LawyerBae24,

The biggest problem with answer choice (e) is that it provides support for the conclusion drawn. Unlike our stimulus, which makes a choice between two possible options without any support, answer choice (e) provides support. The loss of thousands of manufacturing jobs supports the conclusion that the government unemployment figures are inaccurate. Our stimulus, on the other hand, gives no reason to believe that Henning was lying about his support of the proposal instead of Stallworth.

Hope that helps!
Rachael
User avatar
 goingslow
  • Posts: 52
  • Joined: Aug 24, 2021
|
#95100
Hi! What's the issue with (D)?

If we think the flaw is ignoring an obvious alternative explanation (Stallworth could be the one lying), then in (D)'s scenario, it could be Harris's political rivals who are wrong?

Thank you so much!
 Adam Tyson
PowerScore Staff
  • PowerScore Staff
  • Posts: 5153
  • Joined: Apr 14, 2011
|
#95120
In the stimulus, since the proposal did not pass the conclusion is that Henning did not support the proposal, but that overlooks the possibility that it was Stallworth who did not, and Henning actually could have. There was a certain outcome that would be impossible if both conditions were met, so at least one must not have been met, and the author focuses on only one of those two conditions.

In answer D, no specific outcome has been met, and the author doesn't conclude that one condition must not have been met while failing to consider that it could have been the other condition.

You're right that D overlooks the possibility that the rivals are wrong or lying about the Mayor, but that's not an alternative explanation for any particular outcome. There is no claim that "if these two things happened, this third thing would have also happened." It's flawed, but the structure of the flaw is not the same.
User avatar
 goingslow
  • Posts: 52
  • Joined: Aug 24, 2021
|
#95158
Thank you, Adam!!
User avatar
 lsatquestions
  • Posts: 66
  • Joined: Nov 08, 2021
|
#96944
Can you please explain why B is incorrect? There's the opinion of city gov vs. private institution and the author sides with the PI to conclude that it's the city gov to blame for the outcome. Is B incorrect because there's no conditional reasoning for the outcome?
 Luke Haqq
PowerScore Staff
  • PowerScore Staff
  • Posts: 747
  • Joined: Apr 26, 2012
|
#97579
Hi lsatquestions!

Happy to address why answer choice (B) is incorrect.

The description you provided mentioned one flaw in the stimulus--that the author sides with one side over the other. In general, that's an accurate description, but it can be more specific in terms of unpacking why siding with one over the other is a flaw. Namely, it might be a flaw to choose one side over the other if there is no reason to choose one over the other, or if there are better reasons for choosing the option that one did not choose.

This seems to be what is going on in the stimulus. It is saying,

(S) AND (H) :arrow: GA
That is, if S and H both support the proposal, then it will have government approval. And we're given GA, which is in the contrapositive of the above diagram:

GA :arrow: (S) OR (H)
That is, it did not have government approval, which means that it was missing either S's or H's support. The author concludes that H must not have supported it. This is a flaw, because, without more information, we don't know why it couldn't have been S who didn't support it. What is more, it is a conclusion reached "despite [H's] claims to the contrary." Given that extra piece, if anything it'd be more reasonable to conclude that S didn't support it.

This flaw is paralleled in answer choice (A). In that answer choice, there are competing accounts of an accident from Morgan's newspaper report and from the TV news. The author then concludes in favor of one/against the other, but without any additional information that warrants that choice. Additionally, we have something similar to H's claims to the contrary--"Morgan witnessed the accident from his kitchen window." There's no claim that the TV news witnessed it, so if anything, it'd be more reasonable to conclude that Morgan was correct.

Answer choice (B) also involves choosing between two options, but they aren't incompatible options. Both the private institute and the city government can be right about their claims. This aspect of answer choice (B) thus doesn't parallel the flaw in the stimulus. There's additionally no clear reason to prefer either the private institute or the city government. Thus it also doesn't parallel the H's "claims to the contrary" portion of the stimulus, which makes it more reasonable to choose one option over the other.

Get the most out of your LSAT Prep Plus subscription.

Analyze and track your performance with our Testing and Analytics Package.